Difference between revisions of "1994 AHSME Problems/Problem 24"

(Solution)
m (Solution)
Line 5: Line 5:
 
==Solution==
 
==Solution==
 
The minimum range occurs in the set <math>\{7,7,12,12,12\}</math>, so the answer is <math>\boxed{\textbf{(C)}\ 5}</math>
 
The minimum range occurs in the set <math>\{7,7,12,12,12\}</math>, so the answer is <math>\boxed{\textbf{(C)}\ 5}</math>
 +
 +
==See Also==
 +
 +
{{AHSME box|year=1994|num-b=22|num-a=24}}
 +
{{MAA Notice}}

Revision as of 03:36, 28 May 2021

Problem

A sample consisting of five observations has an arithmetic mean of $10$ and a median of $12$. The smallest value that the range (largest observation minus smallest) can assume for such a sample is

$\textbf{(A)}\ 2 \qquad\textbf{(B)}\ 3 \qquad\textbf{(C)}\ 5 \qquad\textbf{(D)}\ 7 \qquad\textbf{(E)}\ 10$

Solution

The minimum range occurs in the set $\{7,7,12,12,12\}$, so the answer is $\boxed{\textbf{(C)}\ 5}$

See Also

1994 AHSME (ProblemsAnswer KeyResources)
Preceded by
Problem 22
Followed by
Problem 24
1 2 3 4 5 6 7 8 9 10 11 12 13 14 15 16 17 18 19 20 21 22 23 24 25 26 27 28 29 30
All AHSME Problems and Solutions

The problems on this page are copyrighted by the Mathematical Association of America's American Mathematics Competitions. AMC logo.png